Somme des quatre carrés par les quaternions — Les-mathematiques.net The most powerful custom community solution in the world

Somme des quatre carrés par les quaternions

Bonjour
Je cherche une jolie preuve (la plus courte possible) montrant qu'il existe, pour $p$ premier, $8(p+1)$ solutions à l'équation $a^2 + b^2 + c^2 + d^2 = p,~ a,b,c,d \in \Bbb Z$.

J'ai trouvé plusieurs preuves sur internet montrant l'existence d'une telle solution, mais aucune dénombrant l'ensemble des solutions...

Avez-vous une référence avec une preuve ? Connaissez-vous vous-même une preuve ?
Si possible, j'aimerais qu'elle utilise les quaternions de Hurwitz, mais je suis preneur de toute preuve quand même !
Je vous remercie par avance !

Réponses

  • La série génératrice de la fonction $r_4$, comptant le nombre de représentations de $n \in \mathbb{N}$ en somme de $4$ carrés (tenant compte des signes et de l'ordre), s'exprime en fonction de la fonction $\vartheta_3$ de Jacobi
    $$\sum_{n=0}^\infty r_4(n) x^n = \vartheta_3^4(x) \quad \left( |x| < 1 \right)$$
    d'où l'on tire
    $$r_4(n) = 8 \sum_{\substack{d \mid n \\ 4 \nmid d}} d.$$

    Référence.

    G. H. Hardy & E. M. Wright, Introduction à la Théorie des Nombres, Trad. F. Sauvageot et Préface de C. Goldstein, Vuibert-Springer, 2007, Théorème 385 p. 404.
  • Merci beaucoup pour cette preuve et sa référence.

    Quelqu'un aurait-il une preuve à base de quaternions ?
  • Oui. Un certain Cherng-tiao Perng, 2011. Chercher sur le net Factorization of Lipschitz Quaternions de l'auteur. Lipschitz car je crois comprendre qu'à l'origine, c'est dû à Lipschitz (1886) et que Perng a simplifié la preuve. A ne pas confondre avec les quaternions de Hurwitz (1896 ?) utilisés dans les preuves ``habituelles'' du théorème de Lagrange. Le théorème de Jacobi figure en proposition 8 du papier que l'on trouve sur le net (prep2011-10-001.pdf)

    Se méfier tout de même car ce n'est pas publié. Ainsi pourquoi pas de trace dans http://www.m-hikari.com/imf/imf-2012/41-44-2012/perngIMF41-44-2012.pdf ?
  • noix de totos : tu devrais dire qu'a priori $r_4(n) = 8 \sum_{\substack{d \mid n \\ 4 \nmid d}} d.$ est un théorème compliqué qui vient du fait que $(\sum_n e^{2i \pi n^2 z})^4$ est une forme modulaire $\in M_2(\Gamma_0(4))$ qui est de dimension finie et de base les séries d'Enseinstein $G_{2,4}(z)= 1+C\sum_{n\ge 1} e^{2i \pi nz} \sum_ {d | n} \chi_4(d) d$ et $G_{2,4}(2z)$
  • Ça ne me semble pas utile à l'intervenant d'évoquer tout ça ici. La référence à Hardy & Wright est bien suffisante dans le contexte.

    Ou alors, il faut tout (re)dire : que représente l'ensemble $\Gamma_0(q)$, ce qu'est une forme modulaire de poids $k$ par rapport à $\Gamma_0(q)$, comment trouve-t-on la dimension de l'espace des formes modulaires de poids $k$, etc.
  • Effectivement, inutile de parler de forme modulaires et de série d'Eisenstein, qui étaient certainement inconnues de Jacobi, qui à ma connaissance est l'auteur de cette formule.
  • @Poirot,
    Petit bémol : on peut penser que Jacobi en connaissait ``un rayon'' : cf les deux trucs attachés. Le premier provient de http://gerard.vdgeer.net/schier06.pdf et le second de Grosswald, Representation of Integers as Sums of Squares. Grosswald fournit plusieurs preuves du theorem 4 (détermination du nombre $r_4(n)$), dont une proche d'une preuve de Jacobi, utilisant la théorie des fonctions elliptiques. Par ailleurs, Jacobi a fourni plusieurs preuves. J'ai voulu en savoir un peu plus et j'ai passé une bonne partie de la journée à essayer de m'y retrouver parmi un certain nombre d'approches ; le bilan, c'est (pour moi) que c'est vachement compliqué de s'y retrouver.87520
    87528
  • @claude : je suis parfaitement d'accord avec ce que tu dis, je voulais juste dire que la théories des formes modulaires, dont Jacobi est certainement l'un des initiateur avec notamment sa fonction thêta, n'existait pas vraiment comme on l'entend aujourd'hui à son époque.
  • $\def\F{\mathbb F}\def\M{\mathbb M}\def\H{\mathbb H}\def\Im{\text{Im}}$Je constate (mieux vaut tard que jamais) que la question initiale concernait la détermination du nombre $r_4(p)$ des solutions entières, $\ge 0$ ou $\le 0$, de $a^2 + b^2 + c^2 + d^2 = p$ pour $p$ premier (impair disons) et pas $r_4(n)$ pour $n$ quelconque. Disons une preuve ``élémentaire'' de $r_4(p) = 8(p+1)$ en utilisant les quaternions. En lisant la réponse de David Speyer in https://mathoverflow.net/questions/84897/proofs-of-jacobis-four-square-theorem, on peut penser qu'il y a quelque chose à gratter. Attention : l'anneau des quaternions utilisé par Speyer n'est pas celui qui vient ci-dessous.

    Voici quelques étapes à assurer (je ne l'ai pas fait !!). Je note $\H = \H_{-1,-1}(\Q)$ l'algèbre des quaternions montée sur $\Q$ avec $i^2 = j^2 = -1$, $ij + ji = 0$, $k = ij$ ...etc.. Et $A$ la $\Z$-algèbre (non commutative) $A = \Z \oplus \Z i \oplus \Z j \oplus \Z k$. Et enfin $N$ la norme sur $\H$ à valeurs dans $\Q_+$, principalement utilisée sur $A$, à valeurs dans $\N$.

    0. Dans $8(p+1)$, tout laisse croire que le $8$ provient de $8 = \#A^\times$ où $A^\times = \{\pm 1, \pm i, \pm j, \pm k\}$ est le groupe des inversibles de $A$ (les éléments de $A$ de norme 1) i.e. le groupe des quaternions. Et le $p+1$, merci Speyer même s'il est dans un contexte légèrement différent, du nombre de droites de $\F_p^2$, cf la suite.

    1. On a un isomorphisme $\H_{-1,-1}(\F_p) \simeq \M_2(\F_p)$ que l'on obtient en utilisant le fait que la conique $u^2 + v^2 + 1 = 0$ admet un zéro sur $\F_p$. Car en notant (encore merci à Speyer, mais c'est classique)
    $$
    I = \left[\matrix {0 & 1\cr -1 &0\cr} \right],\qquad
    J = \left[\matrix {u & v\cr v &-u\cr} \right], \qquad
    K = \left[\matrix {v & -u\cr -u &-v\cr} \right]
    $$
    on a les relations habituelles $I^2 = J^2 = -1$, $K = IJ$ ...etc.. Et $1 = {\rm I}_2, I, J, K$ forment une $\F_p$-base de $\M_2(\F_p)$.

    2. On sait bien que $A$ n'est pas principal (ni à gauche ni à droite). Mais je pense que tout idéal à droite $I$ de $A$ de norme $p$ est principal. En fait, je n'en sais rien mais je fais comme si. Si $z \in A$ vérifie $I = zA$, alors $p = z\overline z = \overline z z \in I$ si bien que $I$ contient $pA$. On peut donc considérer l'idéal $J := I/pA$ de $A/pA \simeq \M_2(\F_p)$. On a $[A : I] = N(z)^2 = p^2$. L'idéal $J$ est donc également d'indice $p^2$ dans $\M_2(\F_p)$.

    Et réciproquement, si $J$ est un idéal à droite de $\M_2(\F_p)$ d'indice $p^2$, il s'écrit $J = I/Ap$ ...etc... Et en croyant que tout va bien se passer, on récupère un idéal $I$ de $A$ d'indice $p^2$, et engendré par un $z$ (défini à un inversible près de $A$) de norme $p$.

    Bref, ici, on croise les doigts. Pendant combien de temps ? Aucune idée. Tout cela pour dire que nous sommes ramenés à l'étude des idéaux (à droite) de $\M_2(\F_p)$ d'indice $p^2$.

    3. On utilise un cas particulier de la correspondance de Morita (je crois que cela s'appelle comme cela). Si $K$ est un corps commutatif (cela sera $\F_p$), on dispose de la correspondance biunivoque entre les idéaux $J$ à droite de $\M_n(K) = \text{End}_K(K^n)$ et les sous-espaces $V$ de $K^n$, assurée par:
    $$
    J \longmapsto V = \sum_{u \in I} \Im\, u, \qquad\qquad\qquad
    J = \{u \in \M_n(K) \mid \Im\, u \subset V\} \leftarrow V
    $$

    4. Pour $n=2$, $K = \F_p$, dans la correspondance ci-dessus, les idéaux d'indice $p^2$ de $\M_2(\F_p)$ correspondraient aux sous-espaces de $\F_p^2$ d'indice $p$ i.e. aux droites de $\F_p^2$. Le voici, en principe, le $p+1$ qui figure dans $r_4(p) = 8(p+1)$.

    5. Il y a pas mal de boulot pour assurer tout cela, en espérant qu'il n'y ait pas trop d'erreurs.
  • $\def\P{\mathbb P}\def\F{\mathbb F}\def\M{\mathbb M}\def\H{\mathbb H}\def\Im{\text{Im}}$Suite (de mon monologue). Soit $p$ un premier impair. Lorsque l'on enchaîne de manière explicite toutes les étapes de mon post précédent i.e. la soi-disant correspondance de Morita + le fait que $\H_{-1,-1}(\F_p) = \M_2(\F_p)$ ...etc... on tombe sur le résultat très simple ci-dessous.

    Je note $C$ la conique projective sur $\F_p$ définie par $u^2 + v^2 + w^2 = 0$. C'est bien connu que cette conique possède un $\F_p$-point et du coup est isomorphe à la droite projective $\P^1(\F_p)$. A fortiori de cardinal $p+1$. Le voilà le $p+1$ qui figure dans $r_4(p) = 8(p+1)$.

    Le résultat très simple réside dans le fait que l'ensemble des idéaux à droite de norme $p$ de $A := \Z \oplus \Z i + \oplus \Z j + \Z k$ est paramétré par cette conique via :
    $$
    C \ni P = (u : v : w) \longmapsto J_P = pA + (ui + vj + wk)A \qquad\qquad (\heartsuit)
    $$
    Ceci s'implémente sans problème si l'on dispose d'un système de Calcul Formel suffisamment avancé
    [color=#000000]> A ;
    Order of Quaternion Algebra with base ring Rational Field defined by i^2=-1, j^2=-1 with coefficient ring Integer Ring
    > Basis(A) ;
    [ 1, i, j, k ]
    > UA ;
    [ -1, 1, -i, i, -j, j, -k, k ]
    [/color]
    
    Une illustration avec $p = 11$.
    [color=#000000]> p := 11 ;
    > Fp := GF(p) ;
    > C<u,v,w> := Conic([Fp| 1,1,1]) ;
    > C ;
    Conic over GF(11) defined by  u^2 + v^2 + w^2
    > Cpoints := Points(C) ;
    > assert #Cpoints eq p+1 ;
    [/color]
    
    Je prends un point $P$ au hasard de la conique et je lui fais le coup de $(\heartsuit)$ pour obtenir l'idéal (à droite) $J_P$ avec sa $\Z$-base
    [color=#000000]> P := Random(Cpoints) ;
    > P ;
    (4 : 4 : 1)
    > u,v,w := Explode(ChangeUniverse(Coordinates(P), Z)) ;
    > u,v,w ;
    4 4 1
    > J := p*A + (u*i + v*j + w*k)*A ;
    > assert Norm(J) eq p ;
    > Basis(J) ;
    [ 11, 11*i, 3 + 10*i + j, 10 + 8*i + k ]
    [/color]
    
    Il se trouve que les outils intègrent la notion de base $\Z$-réduite $(e_1, e_2, e_3, e_4)$ de $J_P$ et également le calcul d'un générateur $z$ de $J_P$ (les deux sont liés). Ce qui va permettre d'expliciter un $z \in A$ de norme $p$ et de visualiser le coup du 8 dans $r_4(p) = 8(p+1)$ : le 8 est à la fois $\#A^\times$ et $2 \times 4$, le 2 pour $\pm$ et le 4 pour $e_1, e_2, e_3, e_4$. En clair :
    $$
    \{ \pm e_1, \pm e_2, \pm e_3, \pm e_4\} = \{\varepsilon z \mid \varepsilon \in A^\times \}, \qquad \qquad A^\times = \{ \pm 1, \pm i, \pm k, \pm k\}
    $$
    [color=#000000]> e := ChangeUniverse(ReducedBasis(J), H) ;
    > e ;
    [ 1 + 3*i - k, -i - j - 3*k, -3 + i - j, 1 - 3*j + k ]
    > {s*ei : ei in e, s in {-1,+1}} ;
    { -1 + 3*j - k, i + j + 3*k, 1 - 3*j + k, -3 + i - j, 1 + 3*i - k, -i - j - 3*k, -1 - 3*i + k, 3 - i + j }
    > ok, z := IsPrincipal(J) ;
    > assert ok ;
    > z ;
    3 - i + j
    > {s*ei : ei in e, s in {-1,+1}} eq {z*epsilon : epsilon in UA} ;
    true
    [/color]
    
  • $\def\H{\mathbb H}\def\Z{\mathbb Z}\def\HZ{\mathbb H(\mathbb Z)}\def\M{\mathbb M}\def\Zn{\Z/n\Z}$Je continue ! Je vais remplacer le premier impair $p$ utilisé jusqu'à maintenant par un entier impair $n$ quelconque. Mais pour l'instant $n \ge 1$ est quelconque (fixé) dans les points 1) 2) 3)

    1) Je considère la famille de matrices de $\M_2(\Z)$ de déterminant $n$, famille indexée par $(d,b)$ où $d \mid n$ et $b \in \Z$,
    $$
    S = S_{d,b} = \left[ \matrix {d & b\cr 0 & n/d} \right] = dE_{11} + bE_{12} + (n/d) E_{22}
    $$
    Si on se limite à $0 \le b \le d-1$, il est clair que cette famille est de cardinal $\fbox {$\sum_{d \mid n} d$}$, nombre que j'aime beaucoup en ce moment.

    2) A chaque matrice $S$, je peux associer son image $L \subset \Z^2$ qui est un réseau d'indice $n$ dans $\Z^2$. Et on obtient ainsi tous les réseaux d'indce $n$ dans $\Z^2$. En nombre $\sum_{d \mid n} d$, le nombre que j'aime beaucoup cette semaine.
    Si on veut faire savant, entre les points 1) et 2), on parle de la correspondance de Morita entre idéaux à droite de $\M_2(\Z)$ et sous-modules de $\Z^2$. En se limitant à l'indice $n$ du côté des sous-modules, l'idéal à droite de $\M_2(\Z)$ qui lui correspond est principal d'indice $n^2$ : et of course, l'idéal à droite correspondant à $L$ est celui engendré par $S$, avec les notations que l'on devine.

    3) Si $L \subset \Z^2$ est d'indice $n$, alors $n\Z^2 \subset L \subset \Z^2$ donc $L/n\Z^2$ est un sous-groupe de $\Z^2/n\Z^2 = (\Zn)^2$ d'indice $n$. Autrement dit, je vais tout passer au dessus de $\Zn$. On obtiendra ainsi la famille des $\sum_{d \mid n} d$ sous-groupes de $(\Zn)^2$ d'indice $n$
    $$
    \langle de_1, (n/d)e_2 + be_1\rangle \qquad d \mid n, \qquad 0 \le b \le d-1
    $$
    Et on pourra continuer à faire du Morita en tombant cette fois dans $\M_2(\Zn)$. On aura donc une correspondance entre les $\sum_{d \mid n} d$ sous-groupes de $(\Zn)^2$ d'indice $n$ et les idéaux à droite (principaux) de $\M_2(\Zn)$ d'indice $n^2$.

    4) Maintenant on suppose $n$ impair. Si bien que l'on dispose d'une interprétation quaternionique de $\M_2(\Zn)$ : $\M_2(\Zn) \simeq \H_{-1,-1}(\Zn)$. Pour réaliser de manière effective cette isomorphie, il suffit de disposer de $u,v \in \Z$ vérifiant $u^2 + v^2 + 1 \equiv 0 \bmod n$. Ce n'est pas difficile à obtenir mais je zappe. On introduit alors $z = 1 - uj - vk$ car c'est lui qui pilote l'interprétation quaternionique :
    $$
    2E_{11} \leftrightarrow z, \qquad 2E_{22} \leftrightarrow \overline z \qquad 2E_{12} \leftrightarrow zi \qquad 2E_{21} \leftrightarrow iz = \overline z i
    $$
    Ici, aussi je zappe mais ce n'est pas bien difficile. Il faut juste déjouer ce que signifie concrètement $\M_2(\Zn) \simeq \H_{-1,-1}(\Zn)$ via un $\Zn$-point de la conique affine $u^2 + v^2 + 1 = 0$. Je vais négliger le 2 qui est un inversible dans $\Zn$ puisque $n$ est impair. L'interprétation quaternionique de la matrice $S_{d,b}$ qui figure dans le point 1) est donc, au facteur 2 près :
    $$
    s = dz + bzi + (n/d) \overline z \qquad \qquad (s \hbox { dépend de } d, b \hbox { mais je ne mentionne pas cette dépendance pour alléger})
    $$
    Tout ceci se passe dans l'anneau des quaternions ordinaires $i^2 = j^2 = k^2 = -1$, $ij = k$ ...etc.. mais au dessus de l'anneau $\Zn$.

    5) Après être descendu de $\Z$ à $\Zn$, on remonte à $\Z$ mais cette fois dans $\HZ$. Et pas dans $\M_2(\Z)$ d'où nous étions partis ! Et on obtient une famille de $\sum_{d\mid n}$ idéaux à droite de $\HZ$ à savoir :
    $$
    I = n\HZ + s\HZ \qquad \qquad (I \hbox { dépend de } d, b \hbox { mais je ne mentionne pas cette dépendance pour alléger})
    $$
    Et la chute, c'est que $I$ est principal : $I = q\HZ$ avec $N(q) = n$. Et on a ainsi obtenu de manière explicite $\sum_{d \mid n} d$ idéaux à droite de $\HZ$ de norme $n$ (pareil que d'indice $n^2$ dans $\HZ$). Sous-entendu : on les obtient tous une (ceux de norme $n$), une et une seule fois (sous-entendu lorsque l'on fait varier $d$ et $b$ là où l'on pense).

    6) Si on tient compte du groupe des inversibles $\HZ^\times = \{\pm 1, \pm i, \pm j, \pm k\}$, on obtient ainsi de manière explicite les $8 \sum_{d\mid n} d$ quaternions $q$ de norme $n$. Et donc les $8\sum_{d \mid n} d$ solutions $(a,b,c,d) \in \Z^4$ de $a^2 + b^2 + c^2 + d^2 = n$.

    7) Une fois que tout ceci est vraiment explicité et ici cela l'est presque, c'est un jeu d'enfants (c'est qui l'enfant ?) de le programmer dans un langage adéquat.
  • $\def\Z{\mathbb Z}\def\HZ{\mathbb H(\mathbb Z)}$1. J'attache une note non terminée.

    2. Je suis à la recherche d'une preuve du résultat suivant : soit $q = a+bi + cj + dk \in \HZ$ un quaternion entier primitif i.e. $\gcd(a,b,c,d) = 1$. Alors l'idéal à droite $q\HZ$ contient un quaternion primitif PUR (pur = sans terme en 1 i.e. de la forme $ui + vj + wk$). Si quelqu'un a une idée, récompense assurée.

    3. Pourquoi je me suis intéressé à ce problème classique de Jacobi ? Pour mieux comprendre ``des choses'' et peut-être pour comprendre un tout petit peu des histoires de graphes de Ramanujan. Cf le dessin attaché tiré du mémoire que l'on trouve in http://www.math.ens.fr/enseignement/archives_pedagogiques.html?type=4 (année 2001-2202). Cf aussi https://www.unine.ch/files/live/sites/math/files/shared/documents/articles/valette22-04-02.pdf

    4. Dans mon post précédent, j'ai beaucoup parlé de $\sigma_1(n) =_{\rm def} \sum_{d \mid n} d$. En liaison avec le théorème de Jacobi $r_4(n) = 8\sum_{d \mid n\atop 4 \not\mid d} d$, nombre de manières d'écrire $n$ comme une somme de 4 carrés dans $\Z^4$. Dans le cas $n$ impair, on a $r_4(n) = 8\sigma_1(n)$. Il est classique de faire intervenir les séries d'Eisenstein disons en une indéterminée $q$ (de manière analytique, $q = e^{2i\pi\tau}$, $\tau $ dans le demi-plan de Poincaré)
    $$
    E_2(q) = 1 - 24 \sum_{n \ge 1} \sigma_1(n) q^n, \qquad\qquad E_{2,N}(q) = N E_2(q^N) - E_2(q)
    $$
    Dans $E_2(q)$, le 24 provient de ${2k \over b_k}$ pour $k = 2$ où $b_k$ est le nombre de Bernoulli d'indice $k$ ($b_2$ vaut $1/6$).
    La série $\Theta$ de $\HZ$:
    $$
    \Theta_{\HZ} = \sum_{z \in \HZ} q^{N(z)} = 1 + \sum_{n \ge 1} r_4(n) q^n = 1 + 8q + 24q^2 + 32 q^3 + \cdots
    $$
    s'exprime en fonction de $E_{2,4}$ : $\Theta_\HZ = {1 \over 3} E_{2,4}$, ce qui est pareil que le théorème de Jacobi $r_4(n) = \cdots$.
    Histoire de voir les séries (ci-dessous $A$ c'est $\HZ$ et $T$ c'est $\Theta_\HZ$)
    [color=#000000]> A ;
    Order of Quaternion Algebra with base ring Rational Field, defined by i^2=-1, j^2=-1 with coefficient ring Integer Ring
    > GramA := GramMatrix(A) ;
    > GramA ;
    [2 0 0 0]
    [0 2 0 0]
    [0 0 2 0]
    [0 0 0 2]
    > L := LatticeWithGram(GramA) ;
    > T<q> := ThetaSeries(L, 10^2 : ExponentConstant := 1/2) ;
    > T + O(q^20) ;
    1 + 8*q + 24*q^2 + 32*q^3 + 24*q^4 + 48*q^5 + 96*q^6 + 64*q^7 + 24*q^8 + 104*q^9 + 144*q^10 + 96*q^11 + 
        96*q^12 + 112*q^13 + 192*q^14 + 192*q^15 + 24*q^16 + 144*q^17 + 312*q^18 + 160*q^19 + O(q^20)
    > r4 := func < n | 8 * &+[d : d in Divisors(n) | not IsDivisibleBy(d,4)] > ;
    > assert &and [r4(n) eq Coefficient(T,n) : n in [1..50]] ;
    [/color]
    
    Et les séries d'Eisenstein
    [color=#000000]
    > E2 := func < q | Parent(T)!Eisenstein(2,q) > ;
    > E2(q) + O(q^20);
    1 - 24*q - 72*q^2 - 96*q^3 - 168*q^4 - 144*q^5 - 288*q^6 - 192*q^7 - 360*q^8 - 312*q^9 - 432*q^10 - 288*q^11 -
        672*q^12 - 336*q^13 - 576*q^14 - 576*q^15 - 744*q^16 - 432*q^17 - 936*q^18 - 480*q^19 + O(q^20)
    > E2minus := func < N | N*E2(q^N) - E2(q) > ;
    > E24<q> := E2minus(4) ;
    > E24 + O(q^20) ;
    3 + 24*q + 72*q^2 + 96*q^3 + 72*q^4 + 144*q^5 + 288*q^6 + 192*q^7 + 72*q^8 + 312*q^9 + 432*q^10 + 288*q^11 + 
        288*q^12 + 336*q^13 + 576*q^14 + 576*q^15 + 72*q^16 + 432*q^17 + 936*q^18 + 480*q^19 + O(q^20)
    > T - 1/3 * E24 ;
    O($.1^20)
    [/color]
    
    L'étude évoquée dans le post précédent a conduit à la petite fonction suivante susceptible d'élaborer, pour $n$ impair, les $\sigma_1(n)$ idéaux (à droite) de $\HZ$ de norme $n$ lorsque l'on fait varier $d$ parmi les diviseurs de $n$ et $b$ dans l'intervalle $0 \le b \le d-1$.
    [color=#000000]NormicRightIdeal := function(n, uv, d, b)   // d | n et u^2 + v^2 + 1 = 0 mod n
      // Retourne l'idéal à droite I = qH(Z) de H(Z), de norme n, associé aux données (u,v, d,b)
      u,v := Explode(uv) ;
      assert IsOdd(n) and  (u^2 + v^2 + 1) mod n eq 0   and  IsDivisibleBy(n,d) ;
      z := 1 - u*j - v*k ;
      s := d*z + b*z*i + ExactQuotient(n,d)*Conjugate(z) ;
      assert Z!Norm(s) mod n eq 0 ;
      I := n*HZ + s*HZ ;
      assert Norm(I) eq n  and Indice(I) eq n^2 ;
      ok, q := IsPrincipal(I) ; assert ok ;
      return I,q ;
    end function ;
    [/color]
    
    88070
  • Salut Claude,

    J'ai un peu regardé ton 2).

    Il suffit de trouver un triplet primitif d'entiers $(u,v,w)$ tel que $a^2+b^2+c^2+d^2$ divise $bu+cv+dw$, $au+dv-cw$, $av+bw-du$ et $aw+cu-bv$.
    En effet, dans ce cas-là,$$q\frac{ bu+cv+dw+i(au+dv-cw)+j(av+bw-du)+k(aw+cu-bv)}{a^2+b^2+c^2+d^2}=ui+vj+wk.$$
    Je ne sais pas si ça peut aider.:-S
  • Salut gai-requin Merci pour ta participation comme on dit. Je n'y suis pas arrivé et je ne sais pas si le résultat est vrai ! Du coup, je me suis fais modeste et j'en ai rabattu. Dans le contexte dans lequel j'étais, j'avais $N(q) = p$ premier impair, plus fort que $q$ primitif, et il suffisait que je trouve un quaternion pur multiple à droite de $q$ dont toutes les composantes ne sont pas divisibles par $p$ (un peu de négation certes mais on n'est pas une négation générale, donc pas trop grave, que je dis). J'attache l'extrait du petit joueur que je suis : $\mathbf A$, c'est $\mathbb H(\mathbb Z)$.

    Autre chose : c'est dingue ce que j'ai appris en regardant ces choses là que je croyais connaître ! Je pense pouvoir attacher une meilleure version que le pdf actuel qui est un véritable b.rdel : quand on a des ennuis avec la technique, impossible de structurer. Je dis pas que la nouvelle version sera bien structurée (en tout cas, pas pire)

    J'ai même appris des choses sur la paramétrisation d'une conique lisse, disons affine, $F(x,y) = 0$ ayant un point rationnel $p_0 = (x_0, y_0)$. Cela te dit quelque chose ? Jusqu'à maintenant, j'introduisais un paramètre $t$ (et j'ai dû le faire une trentaine de fois dans ma vie) et je posais $y-y_0 = t(x-x_0)$ en reportant $y = t(x-x_0) + y_0$ dans $F(x,y) = 0$. Cela te dit quelque chose (bis) ?88130
  • Salut Claude,

    Je trouve ça joli-joli. ;-)

    J'ai un trou : comment calcules-tu l'indice de $\langle p,ui+vj+wk\rangle$ dans $A$ ?
  • Salut Gai-requin

    Ce n'est pas terminé mais j'attache la nouvelle version. J'ai essayé de faire une première page d'introduction pour faire semblant de ne pas être bord.lique. J'ai même fait un résumé (si, si) à la page 6 à la fin de la première partie pour $p$ premier (avant d'attaquer la deuxième partie pour $n$ impair).
    Le calcul de l'indice dont tu parles, je l'ai fait de deux manières différentes aux pages 2-3. Une première manière, c'est du lourd, en déterminant une $\Z$-base de l'idéal. Et une autre un peu moins bourrin.

    Ce qui m'a vachement plu, c'est que pour $n$ impair, des choses se simplifient. Par utilisation d'un machin que j'appelle correspondance de Morita entre idéaux à droite de $M_n(R)$ et sous-modules de $R^n$, $R$ anneau commutatif quelconque. J'ai pris cela en charge car je le connaissais seulement lorsque $R$ est un corps commutatif. D'ailleurs, je ne suis pas persuadé que cela s'appelle vraiment ``correspondance de Morita'' : ce que j'y fais est un truc de bébé.
    J'ai regardé dans Bourbaki Algèbre VIII où il fait la totale sur Morita mais je n'ai pas été foutu d'en extraire ce que je voulais dans le cadre de $R$ et de $M_n(R)$, qui sont Morita équivalents. J'ai pourtant essayé plusieurs fois (de me plonger dans Morita de Bourbaki) mais j'ai capitulé. Et on peut pas tout mettre sur le compte de la chaleur : j'ai fini par penser que lorsque l'on fait la totale, on n'ose pas commencer par un truc de bébé.88140
  • Merci Claude.

    J'avais réussi à calculer l'indice de $qA$ dans $A$ mais pas celui de $I$.
    J'aime bien la deuxième manière. :-)
  • $\def\HZ{\mathbb H(\mathbb Z)}\def\P{\mathbb P}\def\F{\mathbb F}\def\PGL\{\text{PGL}}$Morita : j'aime pas trop raconter n'importe quoi. Je confirme que la correspondance entre idéaux à droite de $M_n(R)$ et sous-modules de $R^n$ est bien liée à l'équivalence de Morita. C'est même vrai si $R$ n'est pas commutatif. J'attache quelques lignes trouvées sur ME in https://math.stackexchange.com/questions/26726/what-are-the-left-and-right-ideals-of-matrix-ring-how-about-the-two-sided-ideal : il s'agit d'idéaux à gauche et il faut donc considérer, pour une matrice, le sous-espace engendré par les lignes (et pas les colonnes). Dans un monde idéal, un exposé sur l'équivalence de Morita devrait commencer par l'exemple du couple d'anneaux $\big(R, M_n(R)\big)$. Et pas par la définition d'équivalence de Morita.

    J'en profite pour faire joujou. De ce qui suit, on pourrait peut-être en tirer une activité pour des petits ? Il s'agit d'écrire un premier $p \ne 2$ comme somme de 4 carrés, $p = a^2 + b^2 + c^2 + d^2$, de toutes les manières possibles dans $\Z^4$. Il y en a $8(p+1)$. Mais ici (et je trouve que ce n'est pas banal), elles sont regroupées par paquets de 8 et donc on fournira seulement $p+1$ paquets, en clair $p+1$ idéaux à droite de $\HZ$. Les $8(p+1)$ sont obtenus par multiplication à droite par $\pm 1, \pm i, \pm j, \pm k$. Je n'ai pas réfléchi à l'utilisation intelligente des automorphismes de la forme quadratique $a^2 + b^2 + c^2 + d^2$ (permuter $a,b,c,d$, remplacer $a,b,c,d$ par $\pm a, \pm b, \pm c, \pm d$) et non plus aux automorphismes de la conique projective $u^2 + v^2 + w^2 = 0 \pmod p$ i.e. à $\PGL_2(\F_p)$.

    J'ai choisi $p = 23$. L'étape 1 de l'activité consiste à ``préparer'' $\F_p$, la conique $u^2 + v^2 + w^2 = 0 \bmod p$ ainsi que $\P^1(\F_p)$
    [color=#000000]> p := 23 ;
    > Fp := GF(p) ;
    > C<u,v,w> := Conic([Fp| 1,1,1]) ;
    > C ;
    Conic over GF(23) defined by u^2 + v^2 + w^2
    > P1<x,y> := Curve(ProjectiveSpace(Fp,1)) ;
    [/color]
    
    La seconde étape consiste à trouver un point sur la conique affine $u^2 + v^2 + 1 = 0$. Mais ce n'est pas difficile car pour environ la moitié des $v \in \F_p$, $-1-v^2$ est un carré. C'est vrai pour tout trinôme $f$ du second degré à coefficients dans un corps fini de caractéristique $\ne 2$ de discriminant non nul : pour environ la moitié des $v$ dans le corps, $f(v)$ est un carré.
    [color=#000000]> // A résoudre : u0^2 + v0^2 + 1 = 0 mod p
    > {* IsSquare(-1-v^2) : v in Fp *} ;
    {* false^^11, true^^12 *}
    > u0,v0 := Explode(uvCoefficients(p)) ;
    > p0 := C![u0,v0] ;
    > p0 ;
    (2 : 8 : 1)
    [/color]
    
    Pour l'instant, c'est abordable pour les petits, je pense. Maintenant cela se complique. C'est l'utilisation de l'idéal à droite $J = p\HZ + (ui + vj + wk)\HZ$ qui va fournir UNE écriture de $p$ comme somme de 4 carrés.
    [color=#000000]ConicRightIdeal := function(P)   // P est un Fp-point de u^2  + v^2 + w^2 = 0
      uvw := Coordinates(P) ;
      u,v,w := Explode(uvw) ;    assert u^2 + v^2 + w^2 eq 0 ;
      p := #Universe(uvw) ;
      u := Z!u ; v := Z!v ; w := Z!w ;
      J := p*A + (u*i + v*j + w*k)*A ;
      assert Norm(J) eq p ;
      return J ;
    end function ;
    [/color]
    
    Ici $p = 23 = 2^2 + 1^2 + 3^2 + (-3)^2$.
    [color=#000000]> I0 := ConicRightIdeal(p0) ;
    > Generateur(I0) ;
    2 + i + 3*j - 3*k
    [/color]
    
    Plus simple est la paramétrisation de la conique $C(\F_p)$ par $\P^1(\F_p)_{(x:y)}$. Hum, plus simple à condition d'y passer plusieurs heures de préparation ??
    [color=#000000]> P1toC := Parametrization(C, p0, P1) ;
    > P1toC : Minimal ;
    (x : y) -> (21*x^2 + 16*x*y + 2*y^2 : 15*x^2 + 19*x*y + 8*y^2 : 22*x^2 + 22*y^2)
    [/color]
    
    On peut lancer la production des $p+1$ paquets. Je montre les $p+1$ points de $\P^1(\F_p)$, pas folichon. Et je leur associe les $p+1$ paquets.
    [color=#000000]> [P : P in Points(P1)] ;
    [ (0 : 1), (1 : 1), (2 : 1), (3 : 1), (4 : 1), (5 : 1), (6 : 1), (7 : 1), (8 : 1), (9 : 1), (10 : 1), (11 : 1), (12 : 1),
     (13 : 1), (14 : 1), (15 : 1), (16 : 1), (17 : 1), (18 : 1), (19 : 1), (20 : 1), (21 : 1), (22 : 1), (1 : 0) ]
    > LeGraalEnIdeaux := [ConicRightIdeal(P1toC(P)) : P in Points(P1)] ;
    [/color]
    
    Le contenu des $p+1$ paquets
    [color=#000000]> [Generateur(I) : I in LeGraalEnIdeaux] ;
    [ -1 - 2*i + 3*j - 3*k, -3 - 2*i + 3*j + k, -3 - i - 2*j + 3*k, 2 - i + 3*j - 3*k, -3 - 3*i - j
    + 2*k, 3 + i + 3*j - 2*k, 1 - 3*i + 2*j - 3*k, -1 + 3*i + 3*j + 2*k, -1 + 3*i + 3*j - 2*k, -1 -
    3*i + 2*j - 3*k, 2 - i - 3*j + 3*k, 1 + 3*i - 3*j + 2*k, -1 - 3*i - 2*j + 3*k, -3 + 2*i + j + 
    3*k, -1 + 3*i - 3*j + 2*k, -1 + 2*i - 3*j + 3*k, -1 + 2*i + 3*j - 3*k, -1 - 2*i - 3*j + 3*k, -2
    + 3*i + 3*j - k, 2 + 3*i + j + 3*k, 3 - i - 2*j + 3*k, -2 - 3*i - j + 3*k, -3 + i + 3*j - 2*k, 
    2 + i + 3*j - 3*k ]
    [/color]
    
    Bonus pour les plus grands : la paramétrisation $\P_1(\F_p) \to C(\F_p)$ est un isomorphisme donc il y a un isomorphisme inverse $C(\F_p) \to \P^1(\F_p)$ qui nécessite DEUX formules relais
    [color=#000000]> BasePoints(P1toC) ;
    {@ @}
    > CtoP1 := Inverse(P1toC) ;
    > CtoP1 : Minimal ;
    (u : v : w) -> (u + 2*w : v + 15*w)
    > BasePoints(CtoP1) ;
    {@ (21 : 8 : 1) @}
    > CtoP1 := Extend(CtoP1) ;
    > CtoP1 : Minimal ;
    (u : v : w) -> (u + 2*w : v + 15*w)
    Alternatives:
    (u : v : w) -> (22*v + 15*w : u + 21*w)
    [/color]
    
    Le coup du BasePoints de la (première) FORMULE $C(\F_p) \to \P^1(\F_p)$ c'est qu'elle ne prend pas en charge le point $(-2 : 8 : 1)$, plus ou moins lié au point initial $p_0$. Il faut donc une seconde formule qui sert de relai. Ici, on apprend que DEUX FORMULES sont nécessaires pour définir UN morphisme. Cela sent encore le cambouis : pourquoi parler formules (polynômes) alors qu'il suffit de parler de morphismes ?
    It's all.88158
Connectez-vous ou Inscrivez-vous pour répondre.
Success message!